Slope of a polynomial function

Click For Summary
The discussion focuses on finding the slope of the polynomial function y = .00002715x^2 - .04934171x + 44.18240907. The derivative of the function is calculated as .0000543x - .04934171, which represents the slope at any point x. To find a specific numerical slope, one must evaluate the derivative at a given x-value. The slope of the line of best fit is confirmed to be .0000543, with a y-intercept of -.04934171. This information is crucial for understanding the behavior of the polynomial function in relation to the data provided.
Cmertin
Messages
57
Reaction score
0

Homework Statement


Slope of: y=.00002715x^2-.04934171x+44.18240907


Homework Equations


d/dx


The Attempt at a Solution


d/dx[.00002715x^2-.04934171x+44.18240907] = .0000543x-.04934171
This is the derivative (slope) of the function though it's looking for a numerical value. It is based off of data that was given. This function is the line of best fit of the data and I need to find the slope of the graph. If you could help me that would be great.

Regards,
Cmertin
 
Physics news on Phys.org
Cmertin said:

Homework Statement


Slope of: y=.00002715x^2-.04934171x+44.18240907


Homework Equations


d/dx


The Attempt at a Solution


d/dx[.00002715x^2-.04934171x+44.18240907] = .0000543x-.04934171
This is the derivative (slope) of the function though it's looking for a numerical value. It is based off of data that was given. This function is the line of best fit of the data and I need to find the slope of the graph. If you could help me that would be great.
When you say "this function" I assume you mean the derivative function, y = .0000543x-.04934171. The slope of a line whose equation is y = mx + b is m, so the slope of your line of best fit is .0000543. The y-intercept of this line is -.04934171.
 
Question: A clock's minute hand has length 4 and its hour hand has length 3. What is the distance between the tips at the moment when it is increasing most rapidly?(Putnam Exam Question) Answer: Making assumption that both the hands moves at constant angular velocities, the answer is ## \sqrt{7} .## But don't you think this assumption is somewhat doubtful and wrong?

Similar threads

Replies
2
Views
1K
Replies
2
Views
1K
Replies
4
Views
1K
  • · Replies 1 ·
Replies
1
Views
1K
Replies
3
Views
2K
  • · Replies 8 ·
Replies
8
Views
3K
Replies
13
Views
4K
  • · Replies 3 ·
Replies
3
Views
2K
  • · Replies 1 ·
Replies
1
Views
2K
Replies
10
Views
6K